2013 AMC 12B Problems/Problem 8

Revision as of 12:20, 6 November 2021 by Equinox8 (talk | contribs) (Solution 2 - Shoelace Theorem: fixed error, previous edit added solution 2)

Problem

Line $l_1$ has equation $3x - 2y = 1$ and goes through $A = (-1, -2)$. Line $l_2$ has equation $y = 1$ and meets line $l_1$ at point $B$. Line $l_3$ has positive slope, goes through point $A$, and meets $l_2$ at point $C$. The area of $\triangle ABC$ is $3$. What is the slope of $l_3$?

$\textbf{(A)}\ \frac{2}{3} \qquad \textbf{(B)}\ \frac{3}{4} \qquad \textbf{(C)}\ 1 \qquad \textbf{(D)}\ \frac{4}{3} \qquad \textbf{(E)}\ \frac{3}{2}$

Solution 1

Line $l_1$ has the equation $y=3x/2-1/2$ when rearranged. Substituting $1$ for $y$, we find that line $l_2$ will meet this line at point $(1,1)$, which is point $B$. We call $\overline{BC}$ the base and the altitude from $A$ to the line connecting $B$ and $C$, $y=-1$, the height. The altitude has length $|-2-1|=3$, and the area of $\triangle{ABC}=3$. Since $A={bh}/2$, $b=2$. Because $l_3$ has positive slope, it will meet $l_2$ to the right of $B$, and the point $2$ to the right of $B$ is $(3,1)$. $l_3$ passes through $(-1,-2)$ and $(3,1)$, and thus has slope $\frac{|1-(-2)|}{|3-(-1)|}=$ $\boxed{\textbf{(B) }\frac{3}{4}}$.

Solution 2 - Shoelace Theorem

We know lines $l_1$ and $l_2$ intersect at $B$, so we can solve for that point: \[3x-2y=1\] Because $y = 1$ we have: \[3x-2(1) = 1\] \[3x-2=1\] \[3x=3\] \[x = 1\]

Thus we have $B = (1,1)$.

We know that the area of the triangle is $3$, so by Shoelace Theorem we have:

\[A = \dfrac{1}{2} |(-2x+y-1) - (-2+x-y)|\] \[A = \dfrac{1}{2} |-2x+y-1+2-x+y|\] \[3 = \dfrac{1}{2} |-2x+y-1+2-x+y|\] \[6 = |-3x+2y+1|.\]

Thus we have two options:

\[6 = -3x+2y+1\] \[5 = -3x+2y\]

or

\[6 = 3x-2y-1\] \[7 = 3x-2y.\]

Now we must just find a point that satisfies $m_{l_3}$ is positive.

Doing some guess-and-check yields, from the second equation:

\[7 = 3x-2y\] \[7 = 3(3)-2(1)\] \[7 = 7\]

so a valid point here is $(3,1)$. When calculated, the slope of $l_3$ in this situation yields $\boxed{\dfrac{3}{4}}$.

Video Solution

https://youtu.be/a-3CAo4CoWc

~Punxsutawney Phil or sugar_rush

See also

2013 AMC 12B (ProblemsAnswer KeyResources)
Preceded by
Problem 7
Followed by
Problem 9
1 2 3 4 5 6 7 8 9 10 11 12 13 14 15 16 17 18 19 20 21 22 23 24 25
All AMC 12 Problems and Solutions

The problems on this page are copyrighted by the Mathematical Association of America's American Mathematics Competitions. AMC logo.png